Difference between revisions of "2020 CIME II Problems/Problem 9"
(→Solution) |
(→Solution) |
||
Line 1: | Line 1: | ||
==Solution== | ==Solution== | ||
− | We can start by finding the number of solution for smaller repeptitions of <math>f</math>. Notice that we can solve <math>f(f(x))=f(x)</math> by applying the functional inverse <math>f^{-1}</math> to both sides as you would to solve any equation: <math>f^{-1}(f(f(x)))=f^{-1}(f(x))\Longrightarrow f(x)=|x|</math> (We put the absolute value bars because we know that taking the inverse of <math>f</math> of both sides involves taking the square root of both sides, and <math>\sqrt{t^2}=|t|</math>). From here, it is easy to see that this equation has <math>4</math> solutions at <math>\pm1</math> and <math>\pm2</math>. We can also try for <math>f(f(f(x)))=f(f(x))</math> (we will solve more methodically here): <cmath>((x^2-2)^2-2)^2-2=(x^2-2)^2-2</cmath> <cmath>(x^2-2)^2-2)^2 = (x^2-2)^2</cmath> <cmath>(x^2-2)^2-2=|x^2-2|</cmath> <cmath>(x^2-2)^2-2=\pm(x^2-2)</cmath> <cmath>(x^2-2)^2=x^2 \textup{ OR } (x^2-2)^2=-x^2+4</cmath>The first equation yeilds <math>4</math> results, and the second equation yields <math>2</math> results for a total of <math>6</math> results. It appears that <math>\underbrace{f(f\cdots f}_{n\textup{ times}}(x))=\underbrace{f(f\cdots f}_{n-1\textup{ times}}(x))</math> bas <math>2n</math> real solutions, giving a total of <math>4040</math> apparent solutions for the original equation. This makes logical sense considering that <math>f</math> is an even polynomial with 2 roots. For a more formal proof, we consider <math>F_n(x)=\underbrace{f(f\cdots f}_{n\textup{ times}}(x))</math>. We are asked to find the number of solutions of the equation in the form <math>F_n(x)=F_{n-1}(x)</math>. Following from how we solved the first simple case, <math>f^{-1}(F_n(x))=f^{-1}(F_{n-1}(x)) = F_{n-1}(x)=|F_{n-2}(x)|</math>. Note that the absolute value branches off in rwo directions: <math>F_{n-1}(x)=\pm F_{n-2}(x)</math>. This would give a total of <math>2\cdot2n=4n</math> real and complex solutions (we multiply by 2 because <math>f</math> is a quadratic, which has 2 total roots). The complex roots come from the negative branches, so there are <math>2\cdot n=2n</math> complex solutions. Therefore, there are a total of <math>2n</math> real roots, which again gives <math>4040</math> roots for the original question. The question asks for <math>\boxed{040}\equiv4040(\mod1000)</math> | + | We can start by finding the number of solution for smaller repeptitions of <math>f</math>. Notice that we can solve <math>f(f(x))=f(x)</math> by applying the functional inverse <math>f^{-1}</math> to both sides as you would to solve any equation: <math>f^{-1}(f(f(x)))=f^{-1}(f(x))\Longrightarrow f(x)=|x|</math> (We put the absolute value bars because we know that taking the inverse of <math>f</math> of both sides involves taking the square root of both sides, and <math>\sqrt{t^2}=|t|</math>). From here, it is easy to see that this equation has <math>4</math> solutions at <math>\pm1</math> and <math>\pm2</math>. We can also try for <math>f(f(f(x)))=f(f(x))</math> (we will solve more methodically here): <cmath>((x^2-2)^2-2)^2-2=(x^2-2)^2-2</cmath> <cmath>(x^2-2)^2-2)^2 = (x^2-2)^2</cmath> <cmath>(x^2-2)^2-2=|x^2-2|</cmath> <cmath>(x^2-2)^2-2=\pm(x^2-2)</cmath> <cmath>(x^2-2)^2=x^2 \textup{ OR } (x^2-2)^2=-x^2+4</cmath>The first equation yeilds <math>4</math> results, and the second equation yields <math>2</math> results for a total of <math>6</math> results. It appears that <math>\underbrace{f(f\cdots f}_{n\textup{ times}}(x))=\underbrace{f(f\cdots f}_{n-1\textup{ times}}(x))</math> bas <math>2n</math> real solutions, giving a total of <math>4040</math> apparent solutions for the original equation. This makes logical sense considering that <math>f</math> is an even polynomial with 2 roots. For a more formal proof, we consider <math>F_n(x)=\underbrace{f(f\cdots f}_{n\textup{ times}}(x))</math>. We are asked to find the number of solutions of the equation in the form <math>F_n(x)=F_{n-1}(x)</math>. Following from how we solved the first simple case, <math>f^{-1}(F_n(x))=f^{-1}(F_{n-1}(x)) = F_{n-1}(x)=|F_{n-2}(x)|</math>. Note that the absolute value branches off in rwo directions: <math>F_{n-1}(x)=\pm F_{n-2}(x)</math>. This would give a total of <math>2\cdot2n=4n</math> real and complex solutions (we multiply by 2 because <math>f</math> is a quadratic, which has 2 total roots). The complex roots come from the negative branches, so there are <math>2\cdot n=2n</math> complex solutions. Therefore, there are a total of <math>2n</math> real roots, which again gives <math>4040</math> roots for the original question. The question asks for <math>\boxed{040}\equiv4040(\mod1000)<cmath></cmath></math>~bhargavakanakapura |
Revision as of 02:19, 30 November 2021
Solution
We can start by finding the number of solution for smaller repeptitions of . Notice that we can solve by applying the functional inverse to both sides as you would to solve any equation: (We put the absolute value bars because we know that taking the inverse of of both sides involves taking the square root of both sides, and ). From here, it is easy to see that this equation has solutions at and . We can also try for (we will solve more methodically here): The first equation yeilds results, and the second equation yields results for a total of results. It appears that bas real solutions, giving a total of apparent solutions for the original equation. This makes logical sense considering that is an even polynomial with 2 roots. For a more formal proof, we consider . We are asked to find the number of solutions of the equation in the form . Following from how we solved the first simple case, . Note that the absolute value branches off in rwo directions: . This would give a total of real and complex solutions (we multiply by 2 because is a quadratic, which has 2 total roots). The complex roots come from the negative branches, so there are complex solutions. Therefore, there are a total of real roots, which again gives roots for the original question. The question asks for ~bhargavakanakapura